Solving Laplacian Equation with u ≤ √x

  • Thread starter Thread starter PeteSampras
  • Start date Start date
  • Tags Tags
    Laplacian
Click For Summary
The discussion centers on solving the Laplacian equation under the condition that u ≤ √x, leading to the conclusion that u must equal zero. A key point of confusion arises regarding the transformation of v(0) to √R in the context of Harnack's inequality. Participants express uncertainty about the algebraic manipulation involved in this substitution. The consensus indicates that the algebra presented is flawed, and there seems to be no clear resolution to the issue. The conversation highlights the complexities involved in applying Harnack's inequality to this problem.
PeteSampras
Messages
43
Reaction score
2

Homework Statement


Problem 2 http://math.mit.edu/~jspeck/18.152_Spring%202017/Exams/Practice%20Midterm%20Exam.pdf
"Let ##u## such that ##Laplacian( u)=0##
Show if ##u \le \sqrt{x}##, then ##u=0##

Homework Equations


At the solution http://math.mit.edu/~jspeck/18.152_Spring%202017/Exams/Practice%20Midterm%20Exam_Solutions.pdf
define ##v=u + \sqrt{R}##

The Attempt at a Solution



equation 10 says, by Harnack
##\frac{R(R-|x|)}{(R+|x|)^2}v(0) \le v(x) \le \frac{R(R+|x|)}{(R-|x|)^2}v(0) ##

but my question is, why in formula 10 change v(0) by sqrt{R} at the left and right side?
## ( \frac{R(R-|x|)}{(R+|x|)^2}-1) \sqrt{R} \le u(x) \le ( \frac{R(R+|x|)}{(R-|x|)^2}-1) \sqrt{R}##

I understand that change ##v(x) \to u(x) + \sqrt{R}##, but, i don't understand why change v(0) by sqrt{R} at the left and right sides.
 
Last edited:
Physics news on Phys.org
PeteSampras said:
why change v(0) by sqrt{R} at the left and right sides.
I agree with you, the algebra is flawed. I see no way to fix it.
 
Question: A clock's minute hand has length 4 and its hour hand has length 3. What is the distance between the tips at the moment when it is increasing most rapidly?(Putnam Exam Question) Answer: Making assumption that both the hands moves at constant angular velocities, the answer is ## \sqrt{7} .## But don't you think this assumption is somewhat doubtful and wrong?

Similar threads

  • · Replies 3 ·
Replies
3
Views
956
  • · Replies 12 ·
Replies
12
Views
2K
  • · Replies 105 ·
4
Replies
105
Views
6K
  • · Replies 5 ·
Replies
5
Views
2K
Replies
3
Views
2K
Replies
4
Views
1K
Replies
1
Views
1K
  • · Replies 4 ·
Replies
4
Views
2K
  • · Replies 1 ·
Replies
1
Views
1K
Replies
2
Views
2K